K
Khách

Hãy nhập câu hỏi của bạn vào đây, nếu là tài khoản VIP, bạn sẽ được ưu tiên trả lời.

AH
Akai Haruma
Giáo viên
29 tháng 7 2020

Lời giải:

Áp dụng BĐT AM-GM cho các số dương ta có:

$\frac{x^3}{x+1}+\frac{x(x+1)}{4}\geq x^2$

$\frac{y^3}{y+1}+\frac{y(y+1)}{4}\geq y^2$

$\frac{z^3}{z+1}+\frac{z(z+1)}{4}\geq z^2$

Cộng theo vế và thu gọn: $P\geq \frac{3(x^2+y^2+z^2)-(x+y+z)}{4}$

Cũng theo BĐT AM-GM: $(x+y+z)^2\leq 3(x^2+y^2+z^2)\Rightarrow x+y+z\leq \sqrt{3(x^2+y^2+z^2)}$

$\Rightarrow 3(x^2+y^2+z^2)-(x+y+z)\geq 3(x^2+y^2+z^2)-\sqrt{3(x^2+y^2+z^2)}=t^2-t$ với $t=\sqrt{3(x^2+y^2+z^2)}\geq \sqrt{3(xy+yz+xz)}\geq 3$

Dễ thấy $t^2-t=t(t-3)+2(t-3)+6=(t+2)(t-3)+6\geq 6$ với $t\geq 3$

Do đó $P\geq \frac{3(x^2+y^2+z^2)-(x+y+z)}{4}\geq \frac{6}{4}=\frac{3}{2}$

Vậy $P_{\min}=\frac{3}{2}$. Dấu "=" xảy ra khi $x=y=z=1$

19 tháng 7 2017

Áp dụng BĐT AM-GM ta có:

\(\frac{\sqrt{1+x^3+y^3}}{xy}\ge\frac{\sqrt{3\sqrt[3]{x^3y^3}}}{xy}=\frac{\sqrt{3xy}}{xy}=\frac{\sqrt{3}}{\sqrt{xy}}\)

Tương tự cho 2 BĐT còn lại ta có:

\(\frac{\sqrt{1+y^3+z^3}}{yz}\ge\frac{\sqrt{3}}{\sqrt{yz}};\frac{\sqrt{1+z^3+x^3}}{xz}\ge\frac{\sqrt{3}}{\sqrt{xz}}\)

Cộng theo vế 3 BĐT trên ta có:

\(M\ge\sqrt{3}\left(\frac{1}{\sqrt{xy}}+\frac{1}{\sqrt{yz}}+\frac{1}{\sqrt{xz}}\right)=\sqrt{3}\cdot\left(\frac{\sqrt{x}}{\sqrt{xyz}}+\frac{\sqrt{y}}{\sqrt{xyz}}+\frac{\sqrt{z}}{\sqrt{xyz}}\right)\)

\(=\sqrt{3}\cdot\frac{\sqrt{x}+\sqrt{y}+\sqrt{z}}{\sqrt{xyz}}\ge\sqrt{3}\cdot\frac{3\sqrt[3]{\sqrt{xyz}}}{1}=3\sqrt{3}\)

Khi \(x=y=z=1\)

11 tháng 2 2017

Câu hỏi của phan tuấn anh - Toán lớp 9 - Học toán với OnlineMath cái này y hệt, tham khảo đi nếu vẫn chưa làm dc thì nhắn cho mk

AH
Akai Haruma
Giáo viên
1 tháng 9 2019

Cách khác:

Áp dụng BĐT Cauchy-Schwarz:

\(P=\frac{x^4}{x+xy}+\frac{y^4}{y+yz}+\frac{z^4}{z+zx}\geq \frac{(x^2+y^2+z^2)^2}{x+y+z+xy+yz+xz}\)

Áp dụng BĐT AM-GM ta có:

\(x^2+y^2+z^2\geq xy+yz+xz(1)\)

\(\Rightarrow 2(x^2+y^2+z^2)\geq 2(xy+yz+xz)\)

\(\Rightarrow 3(x^2+y^2+z^2)\geq (x+y+z)^2\)

\(\Rightarrow (x+y+z)^2\leq 3(x^2+y^2+z^2)\leq (xy+yz+xz)(x^2+y^2+z^2)\leq (x^2+y^2+z^2)^2\)

\(\Rightarrow x+y+z\le x^2+y^2+z^2(2)\)

Từ $(1);(2)$ suy ra:

\(P\geq \frac{(x^2+y^2+z^2)^2}{2(x^2+y^2+z^2)}=\frac{x^2+y^2+z^2}{2}\geq \frac{xy+yz+xz}{2}\geq \frac{3}{2}\)

Vậy $P_{\min}=\frac{3}{2}$

AH
Akai Haruma
Giáo viên
1 tháng 9 2019

Lời giải:

Áp dụng BĐT AM-GM:

\(\frac{x^3}{y+1}+\frac{y+1}{4}+\frac{1}{2}\geq 3\sqrt[3]{\frac{x^3}{y+1}.\frac{y+1}{4}.\frac{1}{2}}=\frac{3x}{2}\)

\(\frac{y^3}{z+1}+\frac{z+1}{4}+\frac{1}{2}\geq \frac{3y}{2}\)

\(\frac{z^3}{1+x}+\frac{1+x}{4}+\frac{1}{2}\geq \frac{3z}{2}\)

Cộng theo vế và thu gọn:

\(\Rightarrow P\geq \frac{5}{4}(x+y+z)-\frac{9}{4}\)

Theo hệ quả quen thuộc của BĐT AM-GM:

\((x+y+z)^2\geq 3(xy+yz+xz)\geq 9\)

\(\Rightarrow x+y+z\geq 3\)

\(\Rightarrow P\geq \frac{5}{4}(x+y+z)-\frac{9}{4}\geq \frac{5}{4}.3-\frac{9}{4}=\frac{3}{2}\)

Vậy $P_{\min}=\frac{3}{2}$ khi $x=y=z=1$

13 tháng 3 2021

Nghiệm nguyên hả?

7 tháng 6 2021

\(\dfrac{x-y}{z^2+1}=\dfrac{x-y}{z^2+xy+yz+zx}=\dfrac{x-y}{z\left(z+y\right)+x\left(z+y\right)}=\dfrac{x-y}{\left(x+z\right)\left(z+y\right)}\)

Tương tự: \(\dfrac{y-z}{x^2+1}=\dfrac{y-z}{\left(x+y\right)\left(x+z\right)}\);\(\dfrac{z-x}{y^2+1}=\dfrac{z-x}{\left(x+y\right)\left(y+z\right)}\)

Cộng vế với vế \(\Rightarrow VT=\dfrac{x-y}{\left(x+z\right)\left(y+z\right)}+\dfrac{y-z}{\left(x+y\right)\left(x+z\right)}+\dfrac{z-x}{\left(x+y\right)\left(y+z\right)}\)

\(=\dfrac{\left(x-y\right)\left(x+y\right)+\left(y-z\right)\left(y+z\right)+\left(z-x\right)\left(z+x\right)}{\left(x+y\right)\left(y+z\right)\left(z+x\right)}\)

\(=\dfrac{x^2-y^2+y^2-z^2+z^2-x^2}{\left(x+y\right)\left(y+z\right)\left(z+x\right)}=0\)(đpcm)

26 tháng 12 2019

Áp dụng BĐT Cô - si cho 3 số không âm:

\(1+x^3+y^3\ge3\sqrt[3]{1.x^3y^3}=3xy\Rightarrow\frac{\sqrt{1+x^3+y^3}}{xy}\ge\frac{\sqrt{3}}{\sqrt{xy}}\)

Tương tự ta có: \(\frac{\sqrt{1+y^3+z^3}}{yz}\ge\frac{\sqrt{3}}{\sqrt{yz}}\);\(\frac{\sqrt{1+z^3+x^3}}{zx}\ge\frac{\sqrt{3}}{\sqrt{zx}}\)

Cộng các vế của các BĐT trên, ta được:

\(\frac{\sqrt{1+x^3+y^3}}{xy}\)\(+\frac{\sqrt{1+y^3+z^3}}{yz}\)\(+\frac{\sqrt{1+z^3+x^3}}{zx}\ge\)\(\frac{\sqrt{3}}{\sqrt{xy}}\)\(+\frac{\sqrt{3}}{\sqrt{yz}}\)\(+\frac{\sqrt{3}}{\sqrt{zx}}\)

Tiếp tục áp dụng Cô - si:

\(BĐT\ge3\sqrt[3]{\frac{\sqrt{3}}{\sqrt{xy}}.\frac{\sqrt{3}}{\sqrt{yz}}.\frac{\sqrt{3}}{\sqrt{zx}}}=3\sqrt{3}\)

Vậy \(\frac{\sqrt{1+x^3+y^3}}{xy}\)\(+\frac{\sqrt{1+y^3+z^3}}{yz}\)\(+\frac{\sqrt{1+z^3+x^3}}{zx}\ge3\sqrt{3}\)

(Dấu "="\(\Leftrightarrow x=y=z=1\))

29 tháng 12 2019

\(x^3+y^3+1=x^3+y^3+xyz\ge xy\left(x+y\right)+xyz=xy\left(x+y+z\right)\)

Tương tự:

\(y^3+z^3+1\ge yz\left(x+y+z\right);z^3+x^3+1\ge zx\left(x+y+z\right)\)

\(\Rightarrow VT\ge\frac{\sqrt{xy\left(x+y+z\right)}}{xy}+\frac{\sqrt{yz\left(x+y+z\right)}}{yz}+\frac{\sqrt{zx\left(x+y+z\right)}}{zx}\)

\(=\sqrt{x+y+z}\left(\frac{1}{\sqrt{xy}}+\frac{1}{\sqrt{yz}}+\frac{1}{\sqrt{zx}}\right)\)

\(\ge\sqrt{3\sqrt[3]{xyz}}\cdot3\sqrt[3]{\frac{1}{\sqrt{xy}\cdot\sqrt{yz}\cdot\sqrt{zx}}}=3\sqrt{3}\)

Dấu "=" xảy ra tại \(x=y=z=1\)